Stay ahead of learning milestones! Enroll in a class over the summer!

G
Topic
First Poster
Last Poster
<KCL wanted, K,L on hypotenuse AB of right isosceles ,AK: KL: LB = 1: 2: \sqrt3
parmenides51   1
N 8 minutes ago by Mathzeus1024
Source: 2015 SPbU finals, grades 10-11 p3 v8 - Saint Petersburg State University School Olympiad
On the hypotenuse $AB$ of an isosceles right-angled triangle $ABC$ such $K$ and $L$ are marked, such that $AK: KL: LB = 1: 2: \sqrt3$. Find $\angle KCL$.
1 reply
parmenides51
Jan 24, 2021
Mathzeus1024
8 minutes ago
Problem3
samithayohan   113
N 12 minutes ago by VideoCake
Source: IMO 2015 problem 3
Let $ABC$ be an acute triangle with $AB > AC$. Let $\Gamma $ be its circumcircle, $H$ its orthocenter, and $F$ the foot of the altitude from $A$. Let $M$ be the midpoint of $BC$. Let $Q$ be the point on $\Gamma$ such that $\angle HQA = 90^{\circ}$ and let $K$ be the point on $\Gamma$ such that $\angle HKQ = 90^{\circ}$. Assume that the points $A$, $B$, $C$, $K$ and $Q$ are all different and lie on $\Gamma$ in this order.

Prove that the circumcircles of triangles $KQH$ and $FKM$ are tangent to each other.

Proposed by Ukraine
113 replies
samithayohan
Jul 10, 2015
VideoCake
12 minutes ago
Incenter and concurrency
jenishmalla   4
N 30 minutes ago by Double07
Source: 2025 Nepal ptst p3 of 4
Let the incircle of $\triangle ABC$ touch sides $BC$, $CA$, and $AB$ at points $D$, $E$, and $F$, respectively. Let $D'$ be the diametrically opposite point of $D$ with respect to the incircle. Let lines $AD'$ and $AD$ intersect the incircle again at $X$ and $Y$, respectively. Prove that the lines $DX$, $D'Y$, and $EF$ are concurrent, i.e., the lines intersect at the same point.

(Kritesh Dhakal, Nepal)
4 replies
jenishmalla
Mar 15, 2025
Double07
30 minutes ago
inequalities
pennypc123456789   0
34 minutes ago
Let $a,b,c$ be positive real numbers . Prove that :
$$\dfrac{(a+b+c)^2}{ab+bc +ac } \ge \dfrac{2ab}{a^2+b^2} + \dfrac{2bc}{b^2+c^2} + \dfrac{2ac}{a^2+c^2} $$
0 replies
pennypc123456789
34 minutes ago
0 replies
Ratio of lengths in right-angled triangle
DylanN   1
N 39 minutes ago by Mathzeus1024
Source: South African Mathematics Olympiad 2021, Problem 2
Let $PAB$ and $PBC$ be two similar right-angled triangles (in the same plane) with $\angle PAB = \angle PBC = 90^\circ$ such that $A$ and $C$ lie on opposite sides of the line $PB$. If $PC = AC$, calculate the ratio $\frac{PA}{AB}$.
1 reply
DylanN
Aug 11, 2021
Mathzeus1024
39 minutes ago
Pythagorean new journey
XAN4   4
N an hour ago by XAN4
Source: Inspired by sarjinius
The number $4$ is written on the blackboard. Every time, Carmela can erase the number $n$ on the black board and replace it with a new number $m$, if and only if $|n^2-m^2|$ is a perfect square. Prove or disprove that all positive integers $n\geq4$ can be written exactly once on the blackboard.
4 replies
XAN4
Yesterday at 3:41 AM
XAN4
an hour ago
wu2481632 Mock Geometry Olympiad problems
wu2481632   14
N an hour ago by bin_sherlo
To avoid clogging the fora with a horde of geometry problems, I'll post them all here.

Day I

Day II

Enjoy the problems!
14 replies
wu2481632
Mar 13, 2017
bin_sherlo
an hour ago
Straight line
uTOPi_a   19
N an hour ago by NerdyNashville
Source: 41-st Vietnamese Mathematical Olympiad 2003
The circles $ C_{1}$ and $ C_{2}$ touch externally at $ M$ and the radius of $ C_{2}$ is larger than that of $ C_{1}$. $ A$ is any point on $ C_{2}$ which does not lie on the line joining the centers of the circles. $ B$ and $ C$ are points on $ C_{1}$ such that $ AB$ and $ AC$ are tangent to $ C_{1}$. The lines $ BM$, $ CM$ intersect $ C_{2}$ again at $ E$, $ F$ respectively. $ D$ is the intersection of the tangent at $ A$ and the line $ EF$. Show that the locus of $ D$ as $ A$ varies is a straight line.
19 replies
uTOPi_a
Aug 28, 2004
NerdyNashville
an hour ago
inequalities
pennypc123456789   1
N an hour ago by Double07
Let \( x,y \) be non-negative real numbers.Prove that :
\[ \sqrt{x^4+y^4 } +(2+\sqrt{2})xy \geq x^2+y^2 \]
1 reply
pennypc123456789
Today at 3:28 AM
Double07
an hour ago
Inspired by old results
sqing   2
N an hour ago by sqing
Source: Own
Let $a,b$ be real numbers such that $  a^3 +b^3+6ab=8 . $ Prove that
$$a+b \leq 2$$Let $a,b$ be real numbers such that $ a^2+b^2+a^3 +b^3+8ab=12 . $ Prove that
$$a+b \leq 2$$Let $a,b$ be real numbers such that $a+b + a^2+b^2+a^3 +b^3+8ab=14 . $ Prove that
$$a+b \leq 2$$
2 replies
sqing
Today at 4:53 AM
sqing
an hour ago
2 var inquality
sqing   2
N an hour ago by sqing
Source: Own
Let $ a,b>0 . $ Prove that
$$\dfrac{1}{a^2+b^2}+\dfrac{1}{4ab} \ge \dfrac{\dfrac{3}{2} +\sqrt 2}{(a+b)^2} $$$$\dfrac{1}{a^3+b^3}+\dfrac{1}{4ab(a+b)}\ge \dfrac{\dfrac{7}{4} +\sqrt 3}{(a+b)^3} $$
2 replies
sqing
4 hours ago
sqing
an hour ago
Problem 1
blug   7
N 2 hours ago by kokcio
Source: Polish Math Olympiad 2025 Finals P1
Find all $(a, b, c, d)\in \mathbb{R}$ satisfying
\[\begin{aligned}
\begin{cases}
    a+b+c+d=0,\\
    a^2+b^2+c^2+d^2=12,\\
    abcd=-3.\\
\end{cases}
\end{aligned}\]
7 replies
blug
Apr 4, 2025
kokcio
2 hours ago
IMO Shortlist 2010 - Problem G1
Amir Hossein   130
N Mar 29, 2025 by LeYohan
Let $ABC$ be an acute triangle with $D, E, F$ the feet of the altitudes lying on $BC, CA, AB$ respectively. One of the intersection points of the line $EF$ and the circumcircle is $P.$ The lines $BP$ and $DF$ meet at point $Q.$ Prove that $AP = AQ.$

Proposed by Christopher Bradley, United Kingdom
130 replies
Amir Hossein
Jul 17, 2011
LeYohan
Mar 29, 2025
IMO Shortlist 2010 - Problem G1
G H J
The post below has been deleted. Click to close.
This post has been deleted. Click here to see post.
SilverBlaze_SY
66 posts
#123
Y by
Let $H$ denote the orthocentre of $\Delta ABC$.
Let $\angle ACB=\theta\implies\angle APB=\theta$, as points $A,B,C,P$ all lie on $(ABC)$.
Also, as $AEHF$ is cyclic, $\angle CAD=\angle EFH=90^{\circ}-\theta\implies\angle AFE=\theta$.
Again, as $H$ is the incentre of $\Delta EFD$, we have $\angle BFD=\angle AFE=\theta$.
Therefore, $AFDP$ is cyclic. Then $\angle AFP=\angle AQP=\theta\implies AQ=AP$, and we're done!
Attachments:
Z K Y
The post below has been deleted. Click to close.
This post has been deleted. Click here to see post.
Jishnu4414l
154 posts
#124
Y by
Very easy even for G1

@above, Ig you need to use directed angles or prove for all possible configurations...
This post has been edited 2 times. Last edited by Jishnu4414l, Apr 26, 2024, 12:11 PM
Z K Y
The post below has been deleted. Click to close.
This post has been deleted. Click here to see post.
P2nisic
406 posts
#125
Y by
Amir Hossein wrote:
Let $ABC$ be an acute triangle with $D, E, F$ the feet of the altitudes lying on $BC, CA, AB$ respectively. One of the intersection points of the line $EF$ and the circumcircle is $P.$ The lines $BP$ and $DF$ meet at point $Q.$ Prove that $AP = AQ.$

Proposed by Christopher Bradley, United Kingdom

$\angle PQF=\angle BQD=180-\angle QBD-\angle BDQ=180-\angle B-\angle A-\angle PBA=\angle C-\angle PBA=\angle PAF$
So $P,Q,F,A$ are cyclic.

$\angle QPA=\angle QFA=180-\angle AFD=180-(180-\angle C)=\angle C=\angle AFE=\angle PQA$
Which gives $AQ=AP$
Attachments:
Z K Y
The post below has been deleted. Click to close.
This post has been deleted. Click here to see post.
Mathandski
738 posts
#126
Y by
Subjective Rating (MOHs) $       $
Attachments:
This post has been edited 1 time. Last edited by Mathandski, Aug 24, 2024, 2:16 AM
Z K Y
The post below has been deleted. Click to close.
This post has been deleted. Click here to see post.
ezpotd
1252 posts
#127
Y by
We claim that $(AFPQ)$ is cyclic, this is trivial by observing $\angle QPA = 180 - \angle BPA = \angle ACB = \angle BFD = \angle AFQ$. Now we see that $\angle AQP = 180 - \angle AFP = 180 - \angle BFD = \angle ACB = 180 - \angle BPA = \angle QPA$, so we are done.
Z K Y
The post below has been deleted. Click to close.
This post has been deleted. Click here to see post.
little-fermat
147 posts
#128
Y by
I have discussed this problem in my EGMO YouTube tutorial ch1 angle chasing practice part
Z K Y
The post below has been deleted. Click to close.
This post has been deleted. Click here to see post.
ehuseyinyigit
808 posts
#129
Y by
I will show the case where point $Q$ is inside of the triangle $ABC$, other situation can be shown similarly.

Since $\angle APQ=\angle C$ and $\angle AFQ=90^{\circ}+\angle CFD=90^{\circ}+\angle CAD=180^{\circ}-\angle C$, we have that the points $A$, $P$, $F$ and $Q$ are concyclic.

Now, observe that we wish to show $\angle APQ=\angle AFP$ which is equilavent to $\angle C=\angle AFP=90^{\circ}-\angle CAD=\angle C$ as desired.
This post has been edited 1 time. Last edited by ehuseyinyigit, Oct 30, 2024, 3:05 PM
Z K Y
The post below has been deleted. Click to close.
This post has been deleted. Click here to see post.
Vedoral
89 posts
#130
Y by
Click to reveal hidden text
Z K Y
The post below has been deleted. Click to close.
This post has been deleted. Click here to see post.
g0USinsane777
44 posts
#131
Y by
Since $APBC$, $BFEC$ and $AFDC$ are cyclic, we have that $\angle AFQ = \angle BFD = \angle ACD = \angle ACB = \angle APQ$, which means that $AFPQ$ is cyclic. Then, because of the above cyclic quadrilaterals, we have that $\angle AQP = \angle AFE = \angle ACB = \angle AFQ = \angle APQ$, giving that triangle $APQ$ is isosceles.
The above proof is for $Q$ outside triangle $ABC$. An analogous proof follows for $Q$ inside $ABC$.
This post has been edited 1 time. Last edited by g0USinsane777, Dec 30, 2024, 6:13 PM
Z K Y
The post below has been deleted. Click to close.
This post has been deleted. Click here to see post.
eg4334
617 posts
#132
Y by
Assume $P$ is closer to $E$, the other config is the same solution. $AFQP$ is cyclis because $\angle QPA = 180 - \angle QFA$. Now $\angle AQP = \angle AFP = \angle AFE = \angle C$ but $\angle APQ = \angle C$ as well.
Z K Y
The post below has been deleted. Click to close.
This post has been deleted. Click here to see post.
Maximilian113
526 posts
#133
Y by
Suppose that $APQF$ forms a non-self-intersecting quadrilateral. Then $\angle APQ = \angle ACB = \angle AFE$ so it suffices to show that $APQF$ is cyclic. But this is easy since $\angle APQ = \angle ACB = 180^\circ - \angle AFD,$ completing the proof. Note that the case when $AQPF$ is cyclic can be dealt with similarly. QED
Z K Y
The post below has been deleted. Click to close.
This post has been deleted. Click here to see post.
thdwlgh1229
17 posts
#135
Y by
Its too easy for IMO Shortlist!
Z K Y
The post below has been deleted. Click to close.
This post has been deleted. Click here to see post.
Ilikeminecraft
330 posts
#137
Y by
Note that $\angle APQ = \angle C = \angle DFB,$ so $AFQP$ is cyclic. Hence, $\angle AQP = \angle AFP = \angle C = \angle APQ$ so we are done.
Z K Y
The post below has been deleted. Click to close.
This post has been deleted. Click here to see post.
blueprimes
325 posts
#138
Y by
We use directed angles throughout this proof. Note that
\[ \angle AFQ = 180^\circ - \angle BFD = 180^\circ - \angle C = 180^\circ - \angle APQ \]so $APQF$ is cyclic. Thus
\[ \angle AQP = \angle ACE = \angle C = \angle APQ \]which finishes.
This post has been edited 1 time. Last edited by blueprimes, Mar 3, 2025, 11:21 PM
Z K Y
The post below has been deleted. Click to close.
This post has been deleted. Click here to see post.
LeYohan
36 posts
#139
Y by
We handle the two cases of the location of $P$:
Let $P_{1}$ be the intersection of $EF$ and $(ABC)$ so that $F$ is between $E$ and $P_{1}$, and $P_{2}$ be the intersection of $EF$ and $(ABC)$ so that $E$ is between $F$ and $P_{2}$.

Case 1: $P \equiv P_{1}$.

Claim: $AFPQ$ is cyclic.

Proof:
We notice that $AFDC$ is cyclic because $\angle AFC = \angle ADC = 90$ implying that $\angle AFQ = \angle C$, and similarly $\angle APQ = \angle C$ so we're done. $\square$

$BFEC$ is cyclic as $\angle BFC = \angle BEC = 90$ meaning that $\angle AFE = \angle C \implies \angle PFA = 180 - \angle C$, and remembering that $AFPQ$ is cyclic we obtain that $\angle AQP = \angle APQ = \angle C$, so $\triangle APQ$ is isosceles with $AP = AQ$, as desired. $\square$

Case 2: $P \equiv P_{2}$.

Claim: $AFPQ$ is cyclic.

Proof:
From the previous case we get that $AFDC$ is cyclic meaning that $\angle BFD = \angle C = \angle QPA$ and we're done. $\square$

From the previous case we also get that $BFEC$ is cyclic so $\angle C = \angle AFP = \angle AQP = \angle QPA$, so $\triangle AQP$ is isosceles with $AQ = AP$ and after covering both configurations we're finally done. $\square$
Z K Y
N Quick Reply
G
H
=
a